a slight bit more clarity
so i definitely understand why d is right and I did choose it. However I try to make anticipation...
Elizabeth25 Wednesday at 04:11PM
  • December 2015 LSAT
  • SEC2
  • Q6
3
Replies
why not b
I get a but I was unnecessary stuck on b as well. what makes b wrong the fact that it said new te...
Elizabeth25 Wednesday at 04:05PM
  • December 2015 LSAT
  • SEC2
  • Q8
1
Reply
Help Needed! (Must Be True - SEC 3 Q16)
I chose A because I thought it made better sense than C since A stated that if you do not have th...
FS101 on October 13 at 09:53PM
  • June 1991 LSAT
  • SEC3
  • Q16
1
Reply
Hi! Could someone please explain why C is corre...
Thank!
DylanMorris on October 13 at 09:36PM
  • September 1998 LSAT
  • SEC3
  • Q21
1
Reply
Why C and not B
I chose B because I thought that both the stimulus and B have to go with a whole -> part fallacy ...
Alec on October 13 at 09:26PM
  • September 1998 LSAT
  • SEC2
  • Q20
1
Reply
Answer B
Could you please explain why B is the correct answer?
yckim2180 on October 13 at 09:24PM
  • September 1998 LSAT
  • SEC2
  • Q19
1
Reply
Why is A better than C?
I'm a little confused about this question. I chose A and am wondering why that was wrong. Thanks!
Alexisnrodriguez93 on October 13 at 09:17PM
  • June 1998 LSAT
  • SEC2
  • Q14
1
Reply
Answer C
I understand why answer D is correct, but could you please explain why answer C is wrong?
yckim2180 on October 13 at 02:44PM
  • June 1998 LSAT
  • SEC4
  • Q6
1
Reply
Isn't D correct too?
Why is D not correct?
hassay18 on October 10 at 03:31PM
  • June 2007 LSAT
  • SEC2
  • Q23
1
Reply
Help please
Could you please explain why the answer is B?
yckim2180 on October 9 at 09:09PM
  • December 1997 LSAT
  • SEC3
  • Q23
1
Reply
Why is D correct?
Why is D correct?
capoleway@gmail.com on October 9 at 09:02PM
  • December 1997 LSAT
  • SEC2
  • Q22
1
Reply
Can someone please explain this?
Could someone please explain this? Thanks
jingjingxiao11111@gmail.com on October 9 at 08:55PM
  • December 1997 LSAT
  • SEC1
  • Q23
1
Reply
Answer C
Could you please explain why C is wrong?
yckim2180 on October 9 at 08:53PM
  • December 1997 LSAT
  • SEC1
  • Q19
1
Reply
Please help
Could you please explain why B is wrong and E is correct?
yckim2180 on October 9 at 08:50PM
  • December 1997 LSAT
  • SEC1
  • Q16
1
Reply
Confusing Explanation for the Correct Answer
I feel like the explanation for the correct answer is very confusing. Can someone please explain ...
hassay18 on October 7 at 02:46PM
  • June 1991 LSAT
  • SEC4
  • Q16
1
Reply
Answer choice B
Why is choice B correct instead of D?
amarachicynthia on October 7 at 02:07PM
  • October 1997 LSAT
  • SEC3
  • Q19
1
Reply
Further explanation
I see where the right answer is coming from I believe, but I was wondering if this could be furth...
AllisonJ on October 7 at 01:58PM
  • October 1997 LSAT
  • SEC4
  • Q21
1
Reply
Explanation of article
Could this article be completely explained because I legit had not idea what it was saying and th...
AllisonJ on October 7 at 01:55PM
  • October 1997 LSAT
  • SEC4
  • Q1
1
Reply
Typo in Correct Answer
The current answer is not proper. I had no idea what it was trying to say. To confirm my suspicio...
Anonymouse on October 6 at 04:39PM
  • October 1997 LSAT
  • SEC2
  • Q11
1
Reply
Answer E
Could you please explain why the answer is E as opposed to A?
yckim2180 on October 6 at 04:07PM
  • June 1997 LSAT
  • SEC1
  • Q6
1
Reply